Art of Problem Solving

2025 AMC 10A Problems/Problem 21: Difference between revisions

Tacos are yummy 1 (talk | contribs)
No edit summary
Stressedpineapple (talk | contribs)
Tag: New redirect
 
(38 intermediate revisions by 25 users not shown)
Line 1: Line 1:
(Problem goes here)
#redirect [[2025 AMC 12A Problems/Problem 15]]
 
==Solution 1==
Let our subset be <imath>\{11,12,13,...,20\}.</imath> If we add any one element from the set <imath>\{1,2,3,...,10\},</imath> we will have to remove at least one element from our current subset. Hence, the size of our set cannot exceed <imath>\boxed{\text{(C) }10}.</imath>
 
~Tacos_are_yummy_1
 
 
==See also==
{{AMC10 box|year=2025|ab=A|num-b=20|num-a=22}}
{{AMC12 box|year=2025|ab=A|num-b=17|num-a=19}}
{{MAA Notice}}

Latest revision as of 02:08, 8 November 2025